Fórum de Matemática
DÚVIDAS? Nós respondemos!

Um Fórum em Português dedicado à Matemática
Data/Hora: 18 abr 2024, 05:58

Os Horários são TMG [ DST ]




Fazer Nova Pergunta Responder a este Tópico  [ 8 mensagens ] 
Autor Mensagem
MensagemEnviado: 30 mar 2012, 18:55 
Offline

Registado: 14 fev 2012, 13:37
Mensagens: 78
Localização: Açores
Agradeceu: 1 vez(es)
Foi agradecido: 0 vez(es)
Peço ajuda, agora, no seguinte integral, passando para coordenadas polares:
\(\int_{0}^{2}\int_{0}^{x}\frac{1}{\sqrt{x^{2}+y^{2}}} dy dx\)


Penso que \(0\leq \theta \leq \frac{\pi }{4}\), certo?
E quanto a r?


Topo
 Perfil  
 
MensagemEnviado: 02 abr 2012, 16:17 
Offline

Registado: 21 jan 2011, 11:31
Mensagens: 947
Localização: Portugal
Agradeceu: 11 vezes
Foi agradecido: 126 vezes
A função integranda é basicamente (1/r).
Logo, ao mudar para coordenadas polares vamos integrar em \(\theta\) e \(\rho\) a função (1/r).r = 1.

Se desenharmos a área de integração, dá-nos um triângulo. A fronteira é a condição y=0, y=x e, importante, x=2.

Se x=2, temos \(\rho cos(\theta) = 2\)

Logo temos a relação extra que falta

\(\rho \leq 2/cos(\theta)\)

_________________
José Sousa
se gostou da resposta, divulgue o fórumdematemática.org

O Binômio de Newton é tão belo como a Vênus de Milo.
O que há é pouca gente para dar por isso.

óóóó---óóóóóó óóó---óóóóóóó óóóóóóóó
(O vento lá fora.)

Álvaro de Campos, 15-1-1928


Topo
 Perfil  
 
MensagemEnviado: 02 abr 2012, 20:45 
Offline

Registado: 14 fev 2012, 13:37
Mensagens: 78
Localização: Açores
Agradeceu: 1 vez(es)
Foi agradecido: 0 vez(es)
Então, como \(1\leq x\leq 2\), vamos ter \(\frac{1}{cos\theta }\leq r\leq \frac{2}{cos \Theta }\).
Vamos ter então que calcular \(\int_{0}^{\frac{\pi }{4}}\int_{\frac{1}{cos \theta }}^{\frac{2}{cos \theta }}1 dr d\theta\).

Pelas minhas "contas", chego ao resultado \(ln(1+\frac{\sqrt{2}}{2})\). No entanto, nas soluções indicadas o resultado é
\(ln(1+sqrt{2})\).
Não consigo detetar onde provavelmente errei. Chegando à parte \(\int_{0}^{\frac{\pi }{4}}\frac{1}{cos \theta } d\Theta\), resolvi por substituição t=cos \(\theta\)
e t' =-sen \(\theta\).
Estará a faltar algum pormenor?
Obrigado!


Topo
 Perfil  
 
MensagemEnviado: 02 abr 2012, 23:15 
Offline

Registado: 05 jan 2011, 12:35
Mensagens: 2235
Localização: Lisboa
Agradeceu: 683 vezes
Foi agradecido: 346 vezes
Meu caro

Apenas um detalhe:

\(0 \leq r\leq \frac{2}{cos \theta }\)

Cumprimentos :)

_________________
João Pimentel Ferreira
 
Partilhe dúvidas e resultados, ajude a comunidade com a sua pergunta!
Não lhe dês o peixe, ensina-o a pescar (provérbio chinês)
Fortalecemos a quem ajudamos pouco, mas prejudicamos se ajudarmos muito (pensamento budista)


Topo
 Perfil  
 
MensagemEnviado: 03 abr 2012, 19:59 
Offline

Registado: 14 fev 2012, 13:37
Mensagens: 78
Localização: Açores
Agradeceu: 1 vez(es)
Foi agradecido: 0 vez(es)
Mas como x varia de 1 a 2, isto não vai influenciar também o raio?
Podia explicar o porquê de \(0\leq r\leq \frac{2}{cos \theta }\)?
Obrigado!


Topo
 Perfil  
 
MensagemEnviado: 04 abr 2012, 00:02 
Offline

Registado: 05 jan 2011, 12:35
Mensagens: 2235
Localização: Lisboa
Agradeceu: 683 vezes
Foi agradecido: 346 vezes
Boas...

Como é que o \(x\) varia entre 1 e 2?

No integral inicial que nos forneceu o \(x\) varia entre 0 e 2

_________________
João Pimentel Ferreira
 
Partilhe dúvidas e resultados, ajude a comunidade com a sua pergunta!
Não lhe dês o peixe, ensina-o a pescar (provérbio chinês)
Fortalecemos a quem ajudamos pouco, mas prejudicamos se ajudarmos muito (pensamento budista)


Topo
 Perfil  
 
MensagemEnviado: 04 abr 2012, 18:53 
Offline

Registado: 14 fev 2012, 13:37
Mensagens: 78
Localização: Açores
Agradeceu: 1 vez(es)
Foi agradecido: 0 vez(es)
Tem toda a razão. Peço desculpa. O engano foi meu quando passei o integral para cá.
Portanto, o exercício inicial é na realidade \(\int_{1}^{2}\int_{0}^{x}\frac{1}{\sqrt{x^{2}+y^{2}}} dy dx\).
.


Topo
 Perfil  
 
MensagemEnviado: 05 abr 2012, 16:38 
Offline

Registado: 05 jan 2011, 12:35
Mensagens: 2235
Localização: Lisboa
Agradeceu: 683 vezes
Foi agradecido: 346 vezes
Então o intervalo que vc utilizou está correto

_________________
João Pimentel Ferreira
 
Partilhe dúvidas e resultados, ajude a comunidade com a sua pergunta!
Não lhe dês o peixe, ensina-o a pescar (provérbio chinês)
Fortalecemos a quem ajudamos pouco, mas prejudicamos se ajudarmos muito (pensamento budista)


Topo
 Perfil  
 
Mostrar mensagens anteriores:  Ordenar por  
Fazer Nova Pergunta Responder a este Tópico  [ 8 mensagens ] 

Os Horários são TMG [ DST ]


Quem está ligado:

Utilizadores a ver este Fórum: Nenhum utilizador registado e 9 visitantes


Criar perguntas: Proibído
Responder a perguntas: Proibído
Editar Mensagens: Proibído
Apagar Mensagens: Proibído
Enviar anexos: Proibído

Pesquisar por:
Ir para:  
cron